Subgroups and LaGrange's Theorem

  • Thread starter Thread starter Obraz35
  • Start date Start date
  • Tags Tags
    Theorem
Click For Summary
H and K are finite subgroups of a group G with relatively prime orders, leading to the conclusion that their intersection can only contain the identity element. By applying LaGrange's theorem, it is established that the order of any subgroup must divide the order of the group. Since H and K have coprime orders, any common subgroup would have an order that divides both, which is only possible if that order is 1. The discussion highlights the importance of understanding subgroup properties and the implications of coprime orders in group theory. Thus, H and K indeed share only the identity element as their intersection.
Obraz35
Messages
29
Reaction score
0
Let H and K be finite subgroups of a group G whose orders are relatively prime. Show H and K have only the identity element in common.

By LaGrange's theorem I know that the orders of H and K must divide the order of G. I have attempted a proof by contradiction but have had no luck arriving at a contradiction. Any ideas?

Thanks.
 
Physics news on Phys.org
You are correct, and you do need Lagrange. But you're asked what the intersection of H and K is. Firstly, is HnK a group?
 
Well, I suppose it would be a group if it contained the identity, g and the inverse of g. This is what I was trying to use to find a contradiction but I'm not sure how this leads to the fact that H and K are relatively prime.
 
You should probably answer Matt's question without the 'if'. If the intersection of H and K is a group, then it's a subgroup of H and K. That tells you something about divisors of the order of H and K.
 
Obraz35 said:
Well, I suppose it would be a group if it contained the identity, g and the inverse of g. This is what I was trying to use to find a contradiction but I'm not sure how this leads to the fact that H and K are relatively prime.

You are attempting to deduce something from the fact that they are relatively prime.

H and K have coprime orders. So, by Lagrange, if L is a subgroup of H and K, then what do you know?
 
Oh, okay. I see it now. Thanks very much.
 
Question: A clock's minute hand has length 4 and its hour hand has length 3. What is the distance between the tips at the moment when it is increasing most rapidly?(Putnam Exam Question) Answer: Making assumption that both the hands moves at constant angular velocities, the answer is ## \sqrt{7} .## But don't you think this assumption is somewhat doubtful and wrong?

Similar threads

  • · Replies 6 ·
Replies
6
Views
2K
  • · Replies 7 ·
Replies
7
Views
2K
  • · Replies 1 ·
Replies
1
Views
1K
  • · Replies 1 ·
Replies
1
Views
2K
  • · Replies 9 ·
Replies
9
Views
2K
  • · Replies 5 ·
Replies
5
Views
3K
  • · Replies 3 ·
Replies
3
Views
2K
  • · Replies 7 ·
Replies
7
Views
2K
  • · Replies 4 ·
Replies
4
Views
1K
  • · Replies 1 ·
Replies
1
Views
2K